Inspired by AMC 12A #22 (2020)

Algebra Level 5

Let c n c_n and d n d_n be integer sequences such that, for all non-negative integers n n , we have

( 1 + 2 ) n = c n + d n 2 \left(1+\sqrt{2} \right)^n = c_n + d_n \sqrt{2}

What is the value of the following expression?

n = 0 c n 2 + 2 d n 2 7 n \sum_{n=0}^{\infty} \dfrac{c^{2}_{n} + 2 d^{2}_{n}}{7^n}


The answer is 3.5.

This section requires Javascript.
You are seeing this because something didn't load right. We suggest you, (a) try refreshing the page, (b) enabling javascript if it is disabled on your browser and, finally, (c) loading the non-javascript version of this page . We're sorry about the hassle.

1 solution

Squaring the given equality, we can find that the rational part Q ( n ) Q(n) of ( 3 + 8 ) n (3+\sqrt{8})^n is equal to c n 2 + 2 d n 2 c^{2}_{n} + 2 d^{2}_{n} .

Knowing that 3 + 8 3+\sqrt{8} is the root of the characteristic polynomial x 2 6 x + 1 = 0 x^2-6x+1 = 0 , we find the recursive relation Q ( n + 2 ) = 6 Q ( n + 1 ) Q ( n ) Q(n+2) = 6Q(n+1) - Q(n) , for Q ( 0 ) = 1 , Q ( 1 ) = 3 Q(0)=1, Q(1)=3 . The exact formula for this recursive Q ( n ) Q(n) can be shown to be

Q ( n ) = ( 3 + 8 ) n + ( 3 8 ) n 2 Q(n) = \dfrac{(3+\sqrt{8})^n + (3-\sqrt{8})^n}{2}

Notice that 3 + 8 7 , 3 8 7 < 1 \left|\frac{3+\sqrt{8}}{7}\right|, \left|\frac{3-\sqrt{8}}{7}\right| <1 . Thus we can apply an infinite sum of a geometric progression to get

n = 0 c n 2 + 2 d n 2 7 n = 1 2 ( 1 1 3 + 8 7 + 1 1 3 8 7 ) = 7 2 ( 1 4 8 + 1 4 + 8 ) = 7 2 . \displaystyle \sum_{n=0}^{\infty} \dfrac{c^{2}_{n} + 2 d^{2}_{n}}{7^n} = \frac{1}{2}\left(\frac{1}{1-\frac{3+\sqrt{8}}{7}}+\frac{1}{1-\frac{3-\sqrt{8}}{7}}\right) = \frac{7}{2}\left(\frac{1}{4-\sqrt{8}}+\frac{1}{4+\sqrt{8}}\right) = \boxed{\frac{7}{2}.}

NICE PROPOSAL :)

Vishwash Kumar ΓΞΩ - 10 months, 2 weeks ago

Or we could use the fact that c n = ( 1 + 2 ) n + ( 1 2 ) n 2 , d n = ( 1 + 2 ) n ( 1 2 ) n 2 2 c_n = \frac{(1+\sqrt{2})^n+(1-\sqrt{2})^n}{2}, d_n = \frac{(1+\sqrt{2})^n - (1-\sqrt{2})^n}{2\sqrt{2}} .

Vikram Sarkar - 10 months, 2 weeks ago

How can you deduct that, Vikram?

Guilherme Dela Corte - 10 months, 2 weeks ago

Using the binomial theorem.

Vikram Sarkar - 10 months, 2 weeks ago

0 pending reports

×

Problem Loading...

Note Loading...

Set Loading...